ATI RN
Pathophysiology Practice Questions Questions
Question 1 of 5
During the home visit of a client with dementia, the nurse notes that an adult daughter persistently corrects her father's misperceptions of reality, even when the father becomes upset and anxious. Which intervention should the nurse teach the caregiver?
Correct Answer: D
Rationale: The correct answer is D: Validation techniques. In dementia care, using validation techniques involves acknowledging the person's feelings and reality, even if it differs from actual events or facts. It helps in reducing the client's anxiety and distress. In this scenario, the daughter persistently correcting her father's misperceptions can escalate his anxiety. Teaching the daughter validation techniques will encourage her to validate her father's feelings and perceptions, ultimately promoting a more supportive and less confrontational environment. Choices A, B, and C are incorrect in this context. While anxiety-reducing measures can be beneficial, the primary issue here is the father's misperceptions being consistently corrected. Positive reinforcement focuses on rewarding desired behaviors, which is not directly related to the situation described. Reality orientation techniques involve constantly reminding the person of the correct time, place, and other details, which may not be suitable for someone with dementia experiencing distress.
Question 2 of 5
Pain in the lower extremities due to peripheral artery disease usually worsens:
Correct Answer: B
Rationale: In peripheral artery disease, pain in the lower extremities worsens with the elevation of the extremity because it diverts blood flow away from the affected area, exacerbating the pain. Choices A, C, and D are incorrect. Resting doesn't increase blood flow, a dependent position doesn't lead to blood pooling in this context, and pain worsening due to touch or massage is not a typical feature of peripheral artery disease.
Question 3 of 5
A patient presents with a sudden onset of severe chest pain radiating to his back. His blood pressure is significantly higher in the right arm than in the left arm. Which of the following conditions is most likely?
Correct Answer: C
Rationale: The correct answer is C, aortic dissection. Aortic dissection is characterized by sudden, severe chest pain that can radiate to the back. The discrepancy in blood pressure between the arms (higher in the right arm) is known as a 'pulse deficit' and is a classic finding in aortic dissection. This condition involves a tear in the inner layer of the aorta, leading to the abnormal flow of blood within the aortic wall. Acute myocardial infarction (choice A) presents with chest pain but typically does not cause discrepancies in blood pressure between arms. Pulmonary embolism (choice B) usually presents with chest pain and difficulty breathing but does not cause unequal blood pressures in the arms. Pericarditis (choice D) can cause chest pain exacerbated by breathing or lying down, but it does not typically lead to differences in blood pressure between arms.
Question 4 of 5
A 30-year-old man has a history of heart transplant and is receiving long-term steroids to prevent rejection. The patient is due for routine vaccines. Attenuated vaccines are contraindicated in this patient because the antigen is:
Correct Answer: A
Rationale: The correct answer is A: live and can cause infection. In patients like the one described with a history of heart transplant and receiving long-term steroids, who are immunocompromised, live attenuated vaccines are contraindicated. Live vaccines contain weakened (attenuated) forms of the virus or bacteria, which can replicate and cause mild infections in healthy individuals but can lead to serious infections in immunocompromised individuals. Choices B, C, and D are incorrect because attenuated vaccines are not mutated, not inactive, and not pathogenic in healthy individuals, respectively.
Question 5 of 5
A 45-year-old woman presents with fatigue, weight gain, and cold intolerance. Her thyroid function tests reveal low T3 and T4 levels and elevated TSH levels. Which of the following is the most likely diagnosis?
Correct Answer: B
Rationale: The scenario describes a 45-year-old woman with fatigue, weight gain, cold intolerance, low T3 and T4 levels, and elevated TSH levels, which are indicative of hypothyroidism. In hypothyroidism, there is decreased production of thyroid hormones (T3 and T4) leading to elevated TSH levels as the pituitary gland tries to stimulate the thyroid gland to produce more hormones. Therefore, the correct answer is hypothyroidism (Choice B). Choice A, Hyperthyroidism, is incorrect as the patient's symptoms and thyroid function tests point towards decreased thyroid hormone levels, which is characteristic of hypothyroidism, not hyperthyroidism. Choice C, Euthyroid sick syndrome, is incorrect because this condition typically presents with normal to low T3 and T4 levels in the setting of non-thyroid illness, unlike the elevated TSH levels observed in this case. Choice D, Subclinical hyperthyroidism, is also incorrect as the patient has low T3 and T4 levels, which is not consistent with hyperthyroidism, whether clinical or subclinical.
Similar Questions
Join Our Community Today!
Join Over 10,000+ nursing students using Nurselytic. Access Comprehensive study Guides curriculum for ATI-RN and 3000+ practice questions to help you pass your ATI-RN exam.
Subscribe for Unlimited Access